- Thu Jan 21, 2016 12:00 am
#38111
Complete Question Explanation
(The complete setup for this game can be found here: lsat/viewtopic.php?t=14969)
The correct answer choice is (E)
This question stipulates that F is evaluated by R. From the second rule, we know that R would also evaluate L, because F and L are evaluated by the same officer. Additionally, from our original diagram we can conclude that the only officer who can evaluate K is T:
With three applicants remaining (H, I, and M), S must evaluate at least two of them in compliance with the last rule. However, the third rule prohibits the same officer from evaluating both H and I, and it also prohibits the same officer from evaluating both M and I. Therefore, S cannot evaluate I, and must evaluate H and M instead:
The question stem asks for how many of the applicants (other than F) is the identity of the officer who evaluates it fully determined. Since exactly five of our variables (not including F) are placed squarely in their respective groups—L, H, M, K, and G—the correct answer choice is (E).
(The complete setup for this game can be found here: lsat/viewtopic.php?t=14969)
The correct answer choice is (E)
This question stipulates that F is evaluated by R. From the second rule, we know that R would also evaluate L, because F and L are evaluated by the same officer. Additionally, from our original diagram we can conclude that the only officer who can evaluate K is T:
With three applicants remaining (H, I, and M), S must evaluate at least two of them in compliance with the last rule. However, the third rule prohibits the same officer from evaluating both H and I, and it also prohibits the same officer from evaluating both M and I. Therefore, S cannot evaluate I, and must evaluate H and M instead:
The question stem asks for how many of the applicants (other than F) is the identity of the officer who evaluates it fully determined. Since exactly five of our variables (not including F) are placed squarely in their respective groups—L, H, M, K, and G—the correct answer choice is (E).
You do not have the required permissions to view the files attached to this post.